1
$\begingroup$

Let $ A $ be an $ \mathcal{H}^1$-measurable subset of $ \mathbb{R} $ and $ \gamma \colon A \subseteq \mathbb{R} \to \ell^\infty $ be a Lipschitz mapping with the Lipschitz constant $ L $. Also, assume that for all $ n \in \mathbb{N} $ and $ \mathcal{H}^1$-a.e. $ t \in A $, $$ \gamma_n'(t) = 0, $$ where $ \gamma_n $ is the $ n^{th} $ component of $ \gamma $.

I want to prove that $$ \lim_{h \to 0}{\frac{\|\gamma(t+h) - \gamma(t)\|_\infty}{\vert h \vert}} = 0, $$ at $ \mathcal{H}^1$-a.e. $ t \in A $.

Any suggestions or ideas are greatly welcomed. Thanks.

Edit: If $ A $ happens to be an interval, the proof is extremely easy because each component is going to be constant. The problem is $ A $ is not necessarily an interval.

$\endgroup$

1 Answer 1

2
$\begingroup$

Is ${\cal H}^1$ one-dimensional Hausdorff measure? So this is just Lebesgue measure?

Then I think the answer is yes, specifically the desired limit is zero at every Lebesgue point of $A$. If $t$ is a Lebesgue point then for any $\epsilon > 0$ we can find $r > 0$ such that $\frac{\mu(A \cap I)}{\mu(I)} \geq 1-\epsilon$ for any interval $I$ centered at $t$ of length at most $2r$. Then any function which has Lipschitz number at most $L$ and whose derivative is zero on $A$ will satisfy $\frac{|f(t+h)-f(t)|}{|h|} \leq 2L\epsilon$ for all $|h| \leq r$. So all components will satisfy this inequality, hence you also get it when you take the sup norm.

$\endgroup$
7
  • $\begingroup$ Thanks Nik. I just have a quick question. How did you conclude that $ \frac{\vert f(t+h) - f(t) \vert}{\vert h \vert} \leq L\epsilon $ for all $ \vert h \vert \leq r $? $\endgroup$
    – Axiom
    Jan 18, 2014 at 23:07
  • $\begingroup$ And yes, $ \mathcal{H}^1 $ is just the 1-dimensional Hausdorff measure on $ \mathbb{R} $ which coincides with the Lebesuge measure on $ \mathbb{R} $. $\endgroup$
    – Axiom
    Jan 18, 2014 at 23:10
  • 1
    $\begingroup$ Extend $f$ to all of ${\bf R}$ without increasing its Lipschitz number, then it is the integral of its derivative which has $L^\infty$ norm at most $L$, and you get the inequality by integrating its derivative. $\endgroup$
    – Nik Weaver
    Jan 18, 2014 at 23:10
  • 1
    $\begingroup$ (If $f$ is complex-valued, then you may have to increase its Lipschitz number by at most a factor of $\sqrt{2}$ when extending it.) $\endgroup$
    – Nik Weaver
    Jan 18, 2014 at 23:11
  • $\begingroup$ Ok, so I think there might be a small problem with your argument. The problem is when you extend $ f $ to the entire set of $ \mathbb{R} $, let's call this extension $ F $, then $ F(t+h) $ is not necessarily equal to $ f(t+h) $ because $ t+h $ is not in $ A $ necessarily. Hence, the estimate $ \frac{\vert f(t+h)-f(t)\vert}{\vert h \vert} \leq 2L\epsilon $ doesn't hold. Am I missing something here possibly? $\endgroup$
    – Axiom
    Jan 20, 2014 at 23:22

Your Answer

By clicking “Post Your Answer”, you agree to our terms of service and acknowledge you have read our privacy policy.

Not the answer you're looking for? Browse other questions tagged or ask your own question.